LSAT and Law School Admissions Forum

Get expert LSAT preparation and law school admissions advice from PowerScore Test Preparation.

 g_lawyered
  • Posts: 213
  • Joined: Sep 14, 2020
|
#92742
Rachael,
Thank you for this!
 sofisofi
  • Posts: 23
  • Joined: Mar 31, 2022
|
#97524
Hi I was wondering if I could get an explanation for why E was incorrect.
thanks!
 Adam Tyson
PowerScore Staff
  • PowerScore Staff
  • Posts: 5191
  • Joined: Apr 14, 2011
|
#97888
The conclusion in the argument is that there is a reason to doubt the correctness of the verdict in cases where the judge fails to control the courtroom and the lawyers start getting out of hand. We need to weaken that by showing that there might NOT be a reason to doubt the verdicts in those cases.

While the selection of jurors based on assessment of their bias is nice, it tells us nothing about what happens AFTER they are selected and the trial begins. Will those jurors be improperly influenced by these lawyers with their wild antics? Or, will they have the resources and wherewithal to see through that nonsense and come to a good verdict? Answer E doesn't tell us about what happens to those allegedly unbiased jurors after the trouble starts in the courtroom.
User avatar
 lsatstudy2023
  • Posts: 13
  • Joined: Jun 28, 2023
|
#102390
Hi, I got this correctly but want to make sure my elimination of A is correct.

I knocked out A because I thought it was out of scope in the sense that it mentioned "incorrect verdicts" and our stimulus was merely talking about " casting doubts on whether or not the verdict was correct". I essentially thought to myself that doubting whether or not something is correct DOES NOT make it incorrect.

I would appreciate your thoughts on this :)
 Rachael Wilkenfeld
PowerScore Staff
  • PowerScore Staff
  • Posts: 1392
  • Joined: Dec 15, 2011
|
#102443
Hi lsatstudy2023,

The big problem with answer choice (A) is that it is about different courtrooms than the stimulus discusses. The stimulus is about courtrooms with judges who are not strict. Answer choice (A) is about courtrooms with judges who ARE strict. It wouldn't impact the argument in the stimulus because it's about a different type of courtroom/judge scenario.

Hope that helps!

Get the most out of your LSAT Prep Plus subscription.

Analyze and track your performance with our Testing and Analytics Package.